LSAT and Law School Admissions Forum

Get expert LSAT preparation and law school admissions advice from PowerScore Test Preparation.

 afinelli
  • Posts: 26
  • Joined: Sep 05, 2011
|
#1820
I'm not clear on why answer choice E is correct for this question (I went with C). Is the key here that the wine example doesn't say that the domestic reduced-price wine did NOT experience an increase in demand - just that the higher priced imported wine increased? At first read, I took it to mean that the reduced-price domestic wine did not increase in sales...
Thanks for any help!
 Nikki Siclunov
PowerScore Staff
  • PowerScore Staff
  • Posts: 1362
  • Joined: Aug 02, 2011
|
#1834
The consequences observed by most wine merchants do not necessarily conflict with the economists' belief, since there is no evidence suggesting that demand for domestic wines did not increase following the price reduction. Just because there is an increase in the sales of imported wines does not mean that there is no comparable increase in the sales of domestic wines as well.

Answer choice (E) shows that an increase in the demand for one product is consistent with an increase in the demand for a competing product, suggesting an increase in the demand for both domestic and imported wines following the price reduction.

If you chose (C), you are assuming that imported wines are superior to domestic wines. Since no comparison is made between the quality of both types of wines, this is not an answer that can explain the paradox.
 afinelli
  • Posts: 26
  • Joined: Sep 05, 2011
|
#1840
Thanks!
 taylorballou
  • Posts: 18
  • Joined: Feb 18, 2017
|
#34263
Hi,

I narrowed down the answer choices for this question down to two contenders, A and E. Originally, I was learning more towards answer choice E but then began to think the juxtaposition of the economists' theory with the merchants' observations implied that while there was an increase in sales for imported wines, the sales of domestic wine remained static (otherwise, why mention it if both increased), so I chose answer A. Now that I'm reading the answer choices again, I suppose A is sort of a cop out answer, which essentially says the economists' theory is wrong in this particular instance.

Thanks,

Taylor
 Ricky_Hutchens
PowerScore Staff
  • PowerScore Staff
  • Posts: 59
  • Joined: Oct 12, 2015
|
#34276
Hi Taylor,

Yeah, A doesn't do much here. We are trying to explain why the Economist theory doesn't appear to work for wine. Even if the Economist rarely study wine, that doesn't explain why demand for wine would behave differently than other things. However, provides us with an explanation that shows wine does behave in the way Economist expect.
 Krwill
  • Posts: 9
  • Joined: Jul 09, 2018
|
#57959
Would this be classified as a Strengthen question?
 Rachael Wilkenfeld
PowerScore Staff
  • PowerScore Staff
  • Posts: 1358
  • Joined: Dec 15, 2011
|
#57980
Hi Krwill,

This is a Resolve the Paradox question. It's similar to strengthen questions in that they both are asking you to help the stimulus. But it's a unique question type as it asks you to help solve a puzzling situation. Your first step in these is always to identify what is the conflict or puzzling situation described in the stimulus.

Hope that helps!
 Krwill
  • Posts: 9
  • Joined: Jul 09, 2018
|
#58101
Thanks for clarifying!!!
 Leela
  • Posts: 63
  • Joined: Apr 13, 2019
|
#64601
Could someone please walk through what a good prephrase for this stimulus would look like?
 Brook Miscoski
PowerScore Staff
  • PowerScore Staff
  • Posts: 418
  • Joined: Sep 13, 2018
|
#64690
Leela,

The knee-jerk response to the situation described in the stimulus is the "snob appeal" economics heuristic--some products are demanded because people think they are more desirable, and dropping the price of one product may make its competitors more desirable if that heuristic is in play.

The answer choice has nothing to do with the most obvious prephrase, because this is a paradox question. I would not spend a great deal of time trying to predict the answers to paradox questions, although having a reaction can help firm up that you understand the problem that needs to be solved--how is it that a price drop in one product will result in its competitor having increased sales?

(E) answers the question in a way we don't expect--by pointing out that increasing overall demand for the type of product can positively impact demand even for the brands that haven't dropped their prices. There is no way that anyone prephrases this, so don't beat yourself up trying to get this kind of prephrase. In my opinion, the correct prephrase is simply "I need a way to let demand for the more expensive product increase."

Get the most out of your LSAT Prep Plus subscription.

Analyze and track your performance with our Testing and Analytics Package.